June 2007 - Sec 1 - Game 1 - Q1

Video Transcript:

0:08
Moving to question one: If the last digit of an acceptable product code is 1, it must
0:14
be true that . . .
0:16
Alright so let's draw out a hypothetical for question one here. What it says is that the
0:24
last digit of an acceptable product code is 1. Well if the last digit is 1, we know for
0:31
certain that we are going to be in scenario two because 1 in scenario one is appearing
0:36
first.
0:38
So since we're in scenario two we know that 2 is going to be first; 4 is going to be second
0:44
because it's exactly twice the first value, or the first digit. Now what do we have left?
0:51
We have 0 and 3. We know that the third digit must less than the fifth digit, which means
0:56
0 must third, leaving 3 for four. So which one of the following must be true
1:03
(A), the first digit is 2. And you notice that must be true so (A) would be the correct
1:09
answer.
1:10
But again just going through the rest of these really quickly just to show you . . .
1:13
(B), the second digit is 0. No, in this case it is 4 and if it must be true it's always
1:21
the case. So one counter example would eliminate B.
1:23
(C), the third digit is 3. Again, the only possibility is that the third digit is
1:28
0 so C is out.
1:30
(D), the fourth digit is 4. Again, the fourth digit is 3 so D does not follow.
1:35
(E), the fourth digit is 0. Again, we know that the fourth digit is 3. So (A) would
1:44
be the correct answer.